Fuerza entre dos cargas puntuales que se mueven paralelas entre sí

Cuando observamos dos cargas puntuales moviéndose paralelas entre sí podemos ver dos fuerzas actuando sobre cada una de las cargas:

  • la fuerza de Coulomb
  • la fuerza magnética ( F = q v × B ) (similar a la fuerza entre dos cables paralelos con la misma corriente)

Sin embargo, si cambiamos al marco de referencia de las cargas, sólo habrá una fuerza, la de Coulomb, y además, la amplitud de esta fuerza debe ser la misma que la de la anterior (la distancia entre las cargas es constante y la la fuerza es independiente de la velocidad). La fuerza resultante debería ser independiente del marco de referencia, pero no lo es y no puedo encontrar la pieza que falta. Sé que un campo magnético es solo un campo eléctrico visto desde un marco de referencia diferente, pero no ayuda en este caso específico.

Encontré que esta pregunta es similar: ¿ dos haces de electrones ejercen diferentes fuerzas entre sí según el marco de referencia? , pero en un escenario de carga, la densidad de carga no tiene significado.

La fuerza de Lorentz, como la he visto definida, tiene en cuenta las contribuciones eléctricas y magnéticas. F Lorentz = q ( mi + v × B ) . Sé que hay otras definiciones, así que aclare cuál está usando. Ver en.wikipedia.org/wiki/Lorentz_force
La pieza que falta es que tienes que transformar correctamente el campo real de una carga en movimiento, que no es lo mismo que el campo de una corriente estacionaria.
@ eepperly16 Ah, así es. Lo que quise decir con la fuerza de Lorentz fue solo la contribución magnética, F = q v × B . Corregiré la pregunta.
No es una respuesta completa, pero vea este video relacionado sobre relatividad y magnetismo youtube.com/watch?v=1TKSfAkWWN0
Ese es exactamente el video del que aprendí que los campos eléctricos y magnéticos son lo mismo visto desde diferentes marcos. Entiendo la situación en el video, pero no hay forma de explicar mi escenario con la contracción de la longitud (¿tal vez la dilatación del tiempo entonces?).
Aquí había una pregunta sobre la violación de la ley 3D de Netwons y la respuesta se encuentra en physics.stackexchange.com/questions/114466/… . Está relacionado con tu pregunta.
Alguna explicación de por qué obtenemos un campo magnético ver aquí
Para que quede claro, la magnitud de la fuerza en el marco del laboratorio no debe ser igual a la magnitud de la fuerza en el marco de las cargas; F = d pag / d t , y mientras d pag será el mismo en este caso (ya que es perpendicular al movimiento relativo de los marcos), d t no se debe a efectos relativistas.

Respuestas (2)

Apenas estoy comenzando a estudiar electromagnetismo, así que salte sobre cualquier error que pueda haber cometido.

La ley de Coulomb solo se cumple para situaciones electrostáticas, por lo que aplicar la ley de Coulomb en el marco de referencia del observador no es válido, ya que las cargas se mueven desde la perspectiva del observador. En cambio, el campo eléctrico y magnético de los sistemas dinámicos viene dado por las cuatro ecuaciones de Maxwell. Incluso para un escenario simple como este, las matemáticas pueden volverse muy complicadas muy rápidamente, pero intentaré explicar la física relevante.

A medida que la partícula se mueve por el espacio, los campos eléctricos y magnéticos en el espacio cambian en función del tiempo. Las ecuaciones de Maxwell nos dicen que los campos magnéticos cambiantes contribuyen al campo eléctrico y los campos eléctricos cambiantes contribuyen al campo magnético. Este vínculo entre los dos campos tiene sentido desde el punto de vista de la relatividad especial, ya que los campos eléctrico y magnético son relativísticamente el mismo campo.

Después de hacer todos los cálculos (lo que involucra un par de ecuaciones diferenciales parciales de segundo orden), podrá calcular las fuerzas eléctricas y magnéticas en cada electrón y sumarlas para encontrar que la fuerza es de hecho equivalente a la fuerza de Coulombic simple de la marco de referencia del electrón.

Gracias, pero como sugiere la página de Wikipedia sobre la ley de Coulomb , la ley es válida cuando las cargas son estacionarias entre sí, como es el caso aquí.
En el marco de referencia del electrón, se cumple la Ley de Coulomb, pero eso no es así para el marco de referencia del observador.
No creo que la fuerza sea "equivalente" en absoluto. La fuerza no es invariante en diferentes marcos de referencia. La fuerza neta se transforma como requiere la relatividad especial.

Si las cargas se mueven paralelas entre sí, en el marco de referencia de una carga, las cargas no se mueven. Entonces el campo magnético es cero y la fuerza es F ¯ = q mi ¯ . Para que la carga esté quieta, la fuerza debe ser cero y, por lo tanto, el campo eléctrico también. Pero si el campo eléctrico no puede ser cero, entonces existe otra fuerza que mantiene las cargas en sus posiciones. En el marco de referencia general tenemos eso (para el cargo): F = 0 > mi ¯ = tu ¯ × B ¯ .

Espero que esto ayude.